Difference between revisions of "2024 AMC 12A Problems/Problem 23"
(→Solution 4(transform)) |
|||
(21 intermediate revisions by 7 users not shown) | |||
Line 51: | Line 51: | ||
~tsun26 | ~tsun26 | ||
− | ==Solution 2 (Another | + | ==Solution 2 (Another Identity)== |
First, notice that | First, notice that | ||
Line 90: | Line 90: | ||
~[https://artofproblemsolving.com/wiki/index.php/User:Reda_mandymath reda_mandymath] | ~[https://artofproblemsolving.com/wiki/index.php/User:Reda_mandymath reda_mandymath] | ||
− | ==Solution 3( | + | ==Solution 3 (Complex Numbers)== |
− | + | Let <math>\theta = \frac{\pi}{16}</math>. Then, | |
+ | <cmath> | ||
+ | y = e^{8i\theta} = e^{\frac{\pi}{2} i} = (\cos \theta + i\sin \theta)^8 = 0 + i. | ||
+ | </cmath> | ||
+ | Expanding by using a binomial expansion, | ||
+ | <cmath> | ||
+ | \Re(y) = \cos^8 \theta - 28 \cos^6 \theta \sin^2 \theta + 70 \cos^4 \theta \sin^4 \theta - 28 \cos^2 \theta \sin^6 \theta + \sin^8\theta =0. | ||
+ | </cmath> | ||
+ | Divide by <math>\cos^8 \theta</math> and notice we can set <math>\frac{\sin \theta}{\cos \theta} = x</math> where <math>x = \tan(\theta)</math>. Then, define <math>f(x)</math> so that | ||
+ | <cmath> | ||
+ | f(x) = 1 - 28 x^2 + 70 x^4 - 28 x^6 + x^8. | ||
+ | </cmath> | ||
− | == | + | Notice that we can have <math>(\cos \theta_k + i \sin \theta_k)^8 = 0 \pm i</math> because we are only considering the real parts. We only have this when <math>k \equiv 1,3 \mod 4</math>, meaning <math>k \equiv 1 \mod 2</math>. This means that we have <math>k = 1,3,5,7,9,11,13,15</math> as unique roots (we get them from <math>k\theta \in [0,\pi]</math>) and by using the fact that <math>\tan(\pi - \theta) = -\tan \theta</math>, we get <cmath>x \in \left\{\tan \theta, -\tan \theta, \tan \left(3 \theta \right), -\tan \left(3 \theta \right), \tan \left(5 \theta \right), -\tan \left(5 \theta \right), \tan \left(7 \theta \right), -\tan \left(7 \theta \right) \right\} </cmath> |
+ | Since we have a monic polynomial, by the Fundamental Theorem of Algebra, | ||
+ | <cmath>f(x) = (x-\tan \theta)(x+\tan \theta) (x-\tan \left(3 \theta \right))(x+\tan \left(3 \theta \right)) (x-\tan \left(5 \theta \right))(x+\tan \left(5 \theta \right))(x-\tan\left(7 \theta \right))(x+\tan \left(7 \theta \right))</cmath> | ||
+ | <cmath>f(x) = (x^2 - \tan^2 \theta)(x^2 - \tan^2 (3\theta))(x^2 - \tan^2 (5\theta))(x^2 - \tan^2 (7\theta)) | ||
+ | </cmath> | ||
+ | Looking at the <math>x^4</math> term in the expansion for <math>f(x)</math> and using vietas gives us | ||
+ | <cmath> | ||
+ | \tan^2 \theta \tan^2 (3\theta) + \tan^2 \theta \tan^2 (5\theta) + \tan^2 \theta \tan^2 (7\theta) + \tan^2 (3\theta) \tan^2 (5\theta) | ||
+ | </cmath> | ||
+ | <cmath> | ||
+ | + \tan^2 (3\theta) \tan^2 (7\theta) + \tan^2 (5\theta) \tan^2 (7\theta) = \frac{70}{1} = 70. | ||
+ | </cmath> | ||
+ | Since <math>\tan\left(\frac{\pi}{2} - \theta\right) = \cot \theta</math> and <math> \tan \theta \cot \theta = 1</math> | ||
+ | <cmath> | ||
+ | \tan^2 \theta \tan^2 (7\theta) = \tan^2 (3\theta) \tan^2 (5\theta) = 1. | ||
+ | </cmath> | ||
+ | Therefore | ||
+ | <cmath> | ||
+ | \tan^2 \theta \tan^2 (3\theta) + \tan^2 \theta \tan^2 (5\theta) + \tan^2 (3\theta) \tan^2 (7\theta) + \tan^2 (5\theta) \tan^2 (7\theta) + 2 = 70. | ||
+ | </cmath> | ||
+ | <cmath> | ||
+ | \tan^2 \theta \tan^2 (3\theta) + \tan^2 \theta \tan^2 (5\theta) + \tan^2 (3\theta) \tan^2 (7\theta) + \tan^2 (5\theta) \tan^2 (7\theta) = \boxed{\textbf{(B) } 68} | ||
+ | </cmath> | ||
+ | |||
+ | ~[https://artofproblemsolving.com/wiki/index.php/User:KEVIN_LIU KEVIN_LIU] | ||
− | + | ==Solution 5 (Transformation)== | |
+ | |||
+ | Set x = <math>\pi/16</math> , 7x = <math>\pi/2</math> - x , | ||
set C7 = <math>cos^2(7x)</math> , C5 = <math>cos^2(5x)</math>, C3 = <math>cos^2(3x)</math>, C= <math>cos^2(x)</math> , S2 = <math>sin^2(2x)</math> , S6 = <math>sin^2(6x), etc.</math> | set C7 = <math>cos^2(7x)</math> , C5 = <math>cos^2(5x)</math>, C3 = <math>cos^2(3x)</math>, C= <math>cos^2(x)</math> , S2 = <math>sin^2(2x)</math> , S6 = <math>sin^2(6x), etc.</math> | ||
Line 114: | Line 151: | ||
~[https://artofproblemsolving.com/wiki/index.php/User:Cyantist luckuso] | ~[https://artofproblemsolving.com/wiki/index.php/User:Cyantist luckuso] | ||
+ | ==Solution 6 (Half angle formula twice)== | ||
+ | So from the question we have: | ||
+ | <cmath>\tan^2 \frac {\pi}{16} \cdot \tan^2 \frac {3\pi}{16} + \tan^2 \frac {\pi}{16} \cdot \tan^2 \frac {5\pi}{16}+\tan^2 \frac {3\pi}{16} \cdot \tan^2 \frac {7\pi}{16}+\tan^2 \frac {5\pi}{16} \cdot \tan^2 \frac {7\pi}{16}</cmath> | ||
+ | |||
+ | |||
+ | <cmath>=(\tan^2\frac{\pi}{16}+\tan^2 \frac{7\pi}{16})(\tan^2\frac{3\pi}{16}+\tan^2 \frac{5\pi}{16})</cmath> | ||
+ | |||
+ | Using <math>\tan^2\frac{\theta}{2}=\frac{1-\cos\theta}{1+\cos\theta}</math> | ||
+ | |||
+ | |||
+ | <cmath>=(\frac{1+\cos\frac{\pi}{8}}{1-\cos\frac{\pi}{8}}+\frac{1+\cos\frac{7\pi}{8}}{1-\cos\frac{7\pi}{8}})(\frac{1+\cos\frac{3\pi}{8}}{1-\cos\frac{3\pi}{8}}+\frac{1+\cos\frac{5\pi}{8}}{1-\cos\frac{5\pi}{8}})</cmath> | ||
+ | |||
+ | Using <math>\cos\theta=-\cos(\pi-\theta)</math> | ||
+ | |||
+ | <cmath>=(\frac{1+\cos\frac{\pi}{8}}{1-\cos\frac{\pi}{8}}+\frac{1-\cos\frac{\pi}{8}}{1+\cos\frac{\pi}{8}})(\frac{1+\cos\frac{3\pi}{8}}{1-\cos\frac{3\pi}{8}}+\frac{1-\cos\frac{3\pi}{8}}{1+\cos\frac{3\pi}{8}})</cmath> | ||
+ | |||
+ | <cmath>=(\frac{(1+\cos\frac{\pi}{8})^2+(1-\cos\frac{\pi}{8})^2}{1-\cos^2\frac{\pi}{8}})(\frac{(1+\cos\frac{3\pi}{8})^2+(1-\cos\frac{3\pi}{8})^2}{1-\cos^2\frac{3\pi}{8}})</cmath> | ||
+ | |||
+ | <cmath>=(\frac{2+2\cos^2\frac{\pi}{8}}{1-\cos^2\frac{\pi}{8}})(\frac{2+2\cos^2\frac{3\pi}{8}}{1-\cos^2\frac{3\pi}{8}})</cmath> | ||
+ | |||
+ | Using <math>\cos^2\frac{\theta}{2}=\frac{1+\cos\theta}{2}</math> | ||
+ | |||
+ | <cmath>=(\frac{2+1+\cos\frac{\pi}{4}}{1-\frac{1+\cos\frac{\pi}{4}}{2}})(\frac{2+1+\cos\frac{3\pi}{4}}{1-\frac{1+\cos\frac{3\pi}{4}}{2}})</cmath> | ||
+ | |||
+ | <cmath>=(\frac{12+2\sqrt{2}}{4-2\sqrt{2}})(\frac{12-2\sqrt{2}}{4+2\sqrt{2}})</cmath> | ||
+ | |||
+ | <cmath>=\frac{136}{2}=\boxed{\textbf{B) }68 }</cmath> | ||
+ | |||
+ | ~ERiccc | ||
+ | ==Solution 7(single formula)== | ||
+ | <cmath>\cot \alpha - \tan \alpha = 2 \cot 2 \alpha \implies \cot^2 \alpha + \tan^2 \alpha = 4 \cot^2 2 \alpha + 2.</cmath> | ||
+ | We use <math>\alpha = \frac {\pi}{16}</math> for <math>(\tan^2\frac{\pi}{16}+\tan^2 \frac{7\pi}{16})(\tan^2\frac{3\pi}{16}+\tan^2 \frac{5\pi}{16}).</math> | ||
+ | |||
+ | <cmath>(\tan^2 \alpha + \cot^2 \alpha)(\tan^2 (\frac{\pi}{4} - \alpha) + \cot^2 (\frac{\pi}{4} - \alpha)) = (4 \cot^2 2 \alpha + 2)(4 \cot^2 (\frac{\pi}{2} - 2\alpha) +2) =</cmath> | ||
+ | <cmath>= 4 \cdot(4+ 2\tan^2 2\alpha + 2\cot^2 2\alpha +1) = 20 + 8 \cdot (4 \cot^2 4 \alpha +2) = 68.\blacksquare</cmath> | ||
+ | '''vladimir.shelomovskii@gmail.com, vvsss''' | ||
+ | |||
+ | ==Solution 8(just do it ✅)== | ||
+ | Since A is too big and E is too small, There is only 3 options left, you can make a guess now, however, estimating the value of it is also okay,you will get 68 for the final answer, I am Rory, I love a girl recently, hope I can persue her successfully | ||
==See also== | ==See also== | ||
{{AMC12 box|year=2024|ab=A|num-b=22|num-a=24}} | {{AMC12 box|year=2024|ab=A|num-b=22|num-a=24}} | ||
{{MAA Notice}} | {{MAA Notice}} |
Latest revision as of 02:09, 21 November 2024
Contents
Problem
What is the value of
Solution 1 (Trigonometric Identities)
First, notice that
Here, we make use of the fact that
Hence,
Note that
Hence,
Therefore, the answer is .
~tsun26
Solution 2 (Another Identity)
First, notice that
Here, we make use of the fact that
Hence,
Therefore, the answer is .
Solution 3 (Complex Numbers)
Let . Then, Expanding by using a binomial expansion, Divide by and notice we can set where . Then, define so that
Notice that we can have because we are only considering the real parts. We only have this when , meaning . This means that we have as unique roots (we get them from ) and by using the fact that , we get Since we have a monic polynomial, by the Fundamental Theorem of Algebra, Looking at the term in the expansion for and using vietas gives us Since and Therefore
Solution 5 (Transformation)
Set x = , 7x = - x , set C7 = , C5 = , C3 = , C= , S2 = , S6 =
First, notice that
Solution 6 (Half angle formula twice)
So from the question we have:
Using
Using
Using
~ERiccc
Solution 7(single formula)
We use for
vladimir.shelomovskii@gmail.com, vvsss
Solution 8(just do it ✅)
Since A is too big and E is too small, There is only 3 options left, you can make a guess now, however, estimating the value of it is also okay,you will get 68 for the final answer, I am Rory, I love a girl recently, hope I can persue her successfully
See also
2024 AMC 12A (Problems • Answer Key • Resources) | |
Preceded by Problem 22 |
Followed by Problem 24 |
1 • 2 • 3 • 4 • 5 • 6 • 7 • 8 • 9 • 10 • 11 • 12 • 13 • 14 • 15 • 16 • 17 • 18 • 19 • 20 • 21 • 22 • 23 • 24 • 25 | |
All AMC 12 Problems and Solutions |
The problems on this page are copyrighted by the Mathematical Association of America's American Mathematics Competitions.